Đến nội dung

Hình ảnh

BĐT -Tuyển tập các bài toán sưu tầm từ Mathlinks.ro

* * * * - 14 Bình chọn tuyển tập sưu tầm.

  • Please log in to reply
Chủ đề này có 74 trả lời

#41
Sagittarius912

Sagittarius912

    Trung úy

  • Thành viên
  • 776 Bài viết

 

Bài 19: Với $a, b, c>0, abc=1$. Chứng minh rằng:
\[\sum_{cyc}{\frac{\sqrt{2a^2+5ab+2b^2}}{1+\sqrt{2a^2+5ab+2b^2}}}\le\frac{3(a+b+c)}{4}\]

em627-oldbeginner

 

Theo bất đẳng thức Cauchy-Schwarz, ta có:

 

$\frac{\sqrt{2a^2+5ab+2b^2}}{1+\sqrt{2a^2+5ab+2b^2}}\le \frac{1}{16}(\frac{\sqrt{2a^2+5ab+2b^2}}{1}+\frac{\sqrt{2a^2+5ab+2b^2}}{\frac{\sqrt{2a^2+5ab+2b^2}}{3}}+\frac{\sqrt{2a^2+5ab+2b^2}}{\frac{\sqrt{2a^2+5ab+2b^2}}{3}}+\frac{\sqrt{2a^2+5ab+2b^2}}{\frac{\sqrt{2a^2+5ab+2b^2}}{3}})$

 

$\Rightarrow \sum \frac{\sqrt{2a^2+5ab+2b^2}}{1+\sqrt{2a^2+5ab+2b^2}}\le \sum \frac{1}{16}(\sqrt{2a^2+5ab+2b^2}+9)$

 

Theo bất đẳng thức AM-GM:

 

$a+b+c\ge 3\sqrt[3]{abc}=3$

 

Do đó ta chỉ cần chứng minh

 

$\sum \sqrt{2a^2+5ab+2b^2}\le 3(a+b+c)$

 

Ta có

 

$2a^2+5ab+2b^2=\frac{9}{4}(a+b)^2-\frac{1}{4}(a-b)^2\le \frac{9}{4}(a+b)^2$

 

$\Rightarrow \sqrt{2a^2+5ab+2b^2} \le \frac{3}{2}(a+b)$

 

$\Rightarrow \sum \sqrt{2a^2+5ab+2b^2} \le 3(a+b+c)$

 

Kết thúc chứng minh.Đẳng thức xảy ra khi  $a=b=c=1$



#42
dark templar

dark templar

    Kael-Invoker

  • Hiệp sỹ
  • 3788 Bài viết

Khởi động lại topic sau 1 thời gian nào ! :)

 

Bài toán 20: Cho đa thức $P(x) \in \mathbb{R}[x]$ có bậc $n>1$ và có đủ $n$ nghiệm phân biệt $x_1;x_2;...;x_{n}$.$P'(x)$ có nghiệm là $y_1;y_2;...;y_{n}$.Chứng minh rằng:

$$(n-1)(x_1^2+x_2^2+...+x_{n}^2)>n(y_1^2+y_2^2+...+y_{n-1}^2)$$


Bài viết đã được chỉnh sửa nội dung bởi dark templar: 04-06-2013 - 11:40

"Do you still... believe in me ?" Sarah Kerrigan asked Jim Raynor - Starcraft II:Heart Of The Swarm.

#43
dark templar

dark templar

    Kael-Invoker

  • Hiệp sỹ
  • 3788 Bài viết


Khởi động lại topic sau 1 thời gian nào ! :)

 

Bài toán 20: Cho đa thức $P(x) \in \mathbb{R}[x]$ có bậc $n>1$ và có đủ $n$ nghiệm phân biệt $x_1;x_2;...;x_{n}$.$P'(x)$ có nghiệm là $y_1;y_2;...;y_{n}$.Chứng minh rằng:

$$(n-1)(x_1^2+x_2^2+...+x_{n}^2)>n(y_1^2+y_2^2+...+y_{n}^2)$$

Lời giải bài toán 20: 

Đầu tiên,ta có:

\[ \left(\frac{n-1}{n}\right)\sum_{i = 1}^{n}x_i =\sum_{i = 1}^{n-1}y_i \]

\[ \left(\frac{n-2}{n}\right)\sum_{1\leq i < j\leq n}x_ix_j =\sum_{1\leq i < j\leq n-1}y_iy_j\,. \]

 

Do đó,vì $ \sum_{i = 1}^{n-1}y_i^2 =\left(\sum_{i = 1}^{n-1}y_i\right)^2-2\sum_{1\leq i < j\leq n-1}y_iy_j $,ta sẽ có:

\[ \sum_{i = 1}^{n-1}y_i^2 =\left(\left(\frac{n-1}{n}\right)\sum_{i = 1}^{n}x_i\right)^2-2\left(\left(\frac{n-2}{n}\right)\sum_{1\leq i < j\leq n}x_ix_j\right) \]

 

Suy ra:

\[ \sum_{i = 1}^{n-1}y_i^2 =\left(\frac{n-1}{n}\right)\sum_{i = 1}^{n}x_i^2-\left(\frac{n-1}{n^2}\left(\sum_{i = 1}^{n}x_i\right)^2-\frac{2}{n}\left(\sum_{1\leq i < j\leq n}x_ix_j\right)\right)\,. \]

 

Ta dễ dàng có BĐT sau,xuất phát từ AM-GM $ 2n\left(\sum_{1\leq i < j\leq n}x_ix_j\right)\leq (n-1)\left(\sum_{i = 1}^{n}x_i\right)^2 $,từ đó cho ta:

\[ \sum_{i = 1}^{n-1}y_i^2\leq\left(\frac{n-1}{n}\right)\sum_{i = 1}^{n}x_i^2\,. \]

 

Dấu bằng xảy ra nếu $x_1=x_2=..=x_{n}$,khi đó các nghiệm $y_{i}$ bằng nhau và bằng với $x_{i}$ với $1 \le i \le n$.

 

==========

Đề mới:

 

Bài toán 21: Cho các số thực $a,b,c$ thỏa mãn $a^2+b^2+c^2=2$.Tìm GTLN và GTNN của $P=\left\lfloor a+b \right\rfloor+\left\lfloor b+c \right\rfloor+\left\lfloor c+a \right\rfloor$.

 

Ký hiệu $\left\lfloor x \right\rfloor$ mang nghĩa là phần nguyên của số thực $x$.

 

Bài toán 22: Cho đa thức $P(x)$ có $\deg P(x)=2$.Giả sử $|P(1)| \le 1;|P(0)| \le 1;|P(-1)| \le 1$.Chứng minh rằng với mọi $x \in [-1;1]$,ta có $|P(x)| \le \frac{5}{4}$.


"Do you still... believe in me ?" Sarah Kerrigan asked Jim Raynor - Starcraft II:Heart Of The Swarm.

#44
Ispectorgadget

Ispectorgadget

    Nothing

  • Quản lý Toán Phổ thông
  • 2946 Bài viết

 

Bài toán 22: Cho đa thức $P(x)$ có $\deg P(x)=2$.Giả sử $|P(1)| \le 1;|P(0)| \le 1;|P(-1)| \le 1$.Chứng minh rằng với mọi $x \in [-1;1]$,ta có $|P(x)| \le \frac{5}{4}$.

Sử dụng công thức nội suy Lagrange ta có $$P(x)=Ax(x+1)+Bx(x-1)+C(x^2-1)$$

 

Cho $x=-1;0;1$ ta có $$B=\frac{P(-1)}{2};C=-P(0);A=\frac{P(1)}{2}$$

 

Từ đó: $$|A|\le \frac{1}{2};\; |B|\le \frac{1}{2};\; |C|\le 1$$

Do đó với mọi x, $|x|\le 1$, ta có $$\begin{align*}
|P(x)| &=|Ax(x+1)+Bx(x-1)+C(x^2-1)| \\
 &\le |A||x(x+1)|+|B||x(x-1)|+|C||(x^2-1)| \\
 &\le 1+|x|-|x|^2 \\
 &= \frac{5}{4}-(|x|-\frac{1}{2})^2 \le \frac{5}{4}.
\end{align*}$$

 

Vậy ta có điều cần chứng minh.


Bài viết đã được chỉnh sửa nội dung bởi Ispectorgadget: 04-06-2013 - 12:12

►|| The aim of life is self-development. To realize one's nature perfectly - that is what each of us is here for. ™ ♫


#45
25 minutes

25 minutes

    Thành viên nổi bật 2015

  • Hiệp sỹ
  • 2795 Bài viết

Bài toán 23 : 

Cho $a,b,c$ là độ dài của 1 tam giác và $a^2+b^2+c^2=12$

Chứng minh rằng : $(a^3+2)(b^3+2)(c^3+2) \leq 872+16abc$

                                                                                                                  -------icp-------


Hãy theo đuổi đam mê, thành công sẽ theo đuổi bạn.



Thảo luận BĐT ôn thi Đại học tại đây


#46
WhjteShadow

WhjteShadow

    Thượng úy

  • Phó Quản lý Toán Ứng dụ
  • 1323 Bài viết

Bài 21:

$\bullet$ GTLN:

Ta có $(a+b+c)^2\leq 3(a^2+b^2+c^2)=6\Rightarrow |a+b+c|\leq \sqrt{6}$, mà $\left\lfloor a+b \right\rfloor \leq a+b$ vậy nên :

$$\left\lfloor a+b \right\rfloor+\left\lfloor b+c \right\rfloor+\left\lfloor c+a \right\rfloor \leq 2\sqrt{6}<5$$

Mà $P$ là số nguyên nên GTLN của nó là $4$, ta chỉ ra 1 bộ $a,b,c$ thoả mãn là $(a;b;c)=(0;1;1)$.

$\bullet$ GTNN:

Do $2=a^2+b^2+c^2\geq a^2+b^2\geq \frac{(a+b)^2}{2}$ nên $a+b\geq -2$ suy ra $\left\lfloor a+b \right\rfloor \geq -2$

Tương tự và cộng lại : $$\left\lfloor a+b \right\rfloor+\left\lfloor b+c \right\rfloor+\left\lfloor c+a \right\rfloor \geq -6$$

Dấu bằng xảy ra chẳng hạn $a=b=c=-\sqrt{\frac{2}{3}}$

Bài 23:

Nhìn có vẻ lừa tình thực ra bài này khá yếu :)) Thậm chí không cần dùng đến giả thiết 3 cạnh tam giác

Đặt $a=2x,b=2y,c=2z$ ta có $x^2+y^2+z^2=3$ và cần chứng minh :

$$(4x^3+1)(4y^3+1)(4z^3+1)\leq 16xyz+109$$

Khai triển và để ý $16(xyz)^3\leq 16xyz$ ta phải chỉ ra :

$$12(xyz)^3+4\sum (xy)^3+\sum x^3\leq 27$$

Điều này luôn đúng khi ta cộng các bđt cùng chiều :

$$\bullet\frac{3}{2}(xyz)^3+4\sum (xy)^3\leq_{AM-GM} \frac{3}{2}(abc)^2+2\sum a^2b^2(a^2+b^2)\\ \leq_{Schur} \frac{(a^2+b^2+c^2)^3}{2}=\frac{27}{2}$$

$$\bullet 6(xyz)^3+x^3+y^3+z^3\leq_{AM-GM} 6(xyz)^{4/3}+x^4+y^4+z^4\leq_{AM-GM} 2\left[(xy)^2+(yz)^2+(xz)^2\right]+x^4+y^4+z^4=9 $$

$$\bullet \frac{9}{2}(xyz)^3\leq \frac{9}{2}$$

Vậy ta có đpcm, đẳng thức xảy ra tại $x=y=z=1$ hay $a=b=c=2$ $\blacksquare$

 


Bài viết đã được chỉnh sửa nội dung bởi WhjteShadow: 04-06-2013 - 17:07

“There is no way home, home is the way.” - Thich Nhat Hanh

#47
dark templar

dark templar

    Kael-Invoker

  • Hiệp sỹ
  • 3788 Bài viết

Đề mới:

 

Bài toán 24: Chứng minh rằng $ \frac{2k+1}{2}+2k^2+k+1 >\sum\limits_{m = k^2+1}^{(k+1)^2 }{\sqrt m },\forall k\in\mathbb{Z}^+ $

 

Bài toán 25: Cho $f(x)=ax^2+bx+c$ với $a \neq 0$ và $|f(x)| \le 1 \quad \forall x \in (0;1]$.Tìm GTLN của $|a|+|b|+|c|$.

 

Mở rộng 1 chút,ta sẽ có 2 bài toán tương tự sau:

 

Bài toán 25.1:Tìm $n \in \mathbb{N^*}$ nhỏ nhất sao cho với $a,b,c$ thực thỏa mãn $|ax^2+bx+c| \le k.\forall x \in [-1;1]$ thì ta có $|a|+|b|+|c| \le nk$.

 

Bài toán 25.2: Tìm $n \in \mathbb{N^*}$ nhỏ nhất sao cho với $a,b,c,d$ thực thỏa mãn $|ax^3+bx^2+cx+d| \le k.\forall x \in [-1;1]$ thì ta có $|a|+|b|+|c|+|d| \le nk$.

 

 

 

 


"Do you still... believe in me ?" Sarah Kerrigan asked Jim Raynor - Starcraft II:Heart Of The Swarm.

#48
dark templar

dark templar

    Kael-Invoker

  • Hiệp sỹ
  • 3788 Bài viết


Đề mới:

 

Bài toán 24: Chứng minh rằng $ \frac{2k+1}{2}+2k^2+k+1 >\sum\limits_{m = k^2+1}^{(k+1)^2 }{\sqrt m },\forall k\in\mathbb{Z}^+ $

 

Bài toán 25: Cho $f(x)=ax^2+bx+c$ với $a \neq 0$ và $|f(x)| \le 1 \quad \forall x \in (0;1]$.Tìm GTLN của $|a|+|b|+|c|$.

Lời giải bài toán 24:

Từ BĐT $ \sqrt{k^2+j}<k+{j\over 2k} $ với $j,k>0$,ta có:

$ S=\sum_{j=1}^{2k+1}\sqrt{k^2+j}=\left(\sum_{j=1}^{2k}\sqrt{k^2+j}\right)+k+1<\left(\sum_{j=1}^{2k}\left(k+{j\over 2k}\right)\right)+k+1 $

$ S<2k^2+{1\over 2k}\cdot{2k(2k+1)\over 2}+k+1 $

$ S<2k^2+k+1+{2k+1\over 2} $

 

Lời giải bài toán 25: 

Đặt $ A=f\left(\frac{1}{2} \right)=\frac{a}{4}+\frac{b}{2}+c; B=f(1)=a+b+c ;C=f(0)=c$.

$ \Rightarrow |A|\le 1, |B|\le 1, |C|\le 1 $

 

Do đó:

$ a=-4A+2B+2C $

$ b=4A-B-3C $

$c=C$

$ \Rightarrow |a|+|b|+|c| $

$ =|-4A+2B+2C|+|4A-B-3C|+|C| \le 4|A|+2|B|+2|C|+4|A|+|B|+3|C|\le 17 $

 

Đẳng thức xảy ra với $f(x)=8x^2-8x+1$.

 

====================

Đề mới:

 

Bài toán 26: Với mỗi $i \in \{1;2;...;n \}(n \in \mathbb{N})$,giả sử ta có số phức $z_{i}$ thỏa $|z_{i}|=1$. Cho $\sum_{i=1}^{n}z_{i}=0$. Nếu $z \in \mathbb{C}$,chứng minh rằng $ n\leq\sum_{i = 1}^{n}\left|z-z_{i}\right| $

 

Bài toán 27: Chứng minh với mọi số tự nhiên $n$,ta có BĐT $\sum_{k=1}^{n^2}\left\{\sqrt{k}\right\}\le\frac{n^2-1}{2}. $

-127-


Bài viết đã được chỉnh sửa nội dung bởi dark templar: 09-06-2013 - 21:22

"Do you still... believe in me ?" Sarah Kerrigan asked Jim Raynor - Starcraft II:Heart Of The Swarm.

#49
Oral1020

Oral1020

    Thịnh To Tướng

  • Thành viên
  • 1225 Bài viết

Bài toán 28: Cho $a,b,c,d >0$ thỏa mãn $a+b+c+d=1$.Chứng minh rằng:
$\sum \dfrac{a}{b} \ge \sum \dfrac{1-b}{1-a}$
Có thể tổng quát được với $n$ biến được không ?

Spoiler


"If I feel unhappy,I do mathematics to become happy.


If I feel happy,I do mathematics to keep happy."

Alfréd Rényi

Hình đã gửi


#50
dark templar

dark templar

    Kael-Invoker

  • Hiệp sỹ
  • 3788 Bài viết


Bài toán 26: Với mỗi $i \in \{1;2;...;n \}(n \in \mathbb{N})$,giả sử ta có số phức $z_{i}$ thỏa $|z_{i}|=1$. Cho $\sum_{i=1}^{n}z_{i}=0$. Nếu $z \in \mathbb{C}$,chứng minh rằng $ n\leq\sum_{i = 1}^{n}\left|z-z_{i}\right| $

 

Bài toán 27: Chứng minh với mọi số tự nhiên $n$,ta có BĐT $\sum_{k=1}^{n^2}\left\{\sqrt{k}\right\}\le\frac{n^2-1}{2}. $

-127-

Lời giải bài toán 26:

Nếu $z=0$ thì $ \sum |z-z_i| =\sum |z_i| = n $.

 

Nếu $z \ne 0$ thì đặt $t=\frac{1}{z}$.Để ý rằng $ z_i =\frac{1}{\overline{z_i}} $ và $|\overline{z_{i}}|=1$,do $|z_{i}|=1$.Do đó:

$ \sum |z-z_i| =\sum\left |\frac{1}{t}-\frac{1}{\overline{z_i}}\right | =\sum\left |\frac{t-\overline{z_i}}{t\overline{z_i}}\right | =\frac{1}{|t|}\sum |t-\overline{z_i}|\geq\frac{1}{|t|}|\sum (t-\overline{z_i})| =\frac{1}{|t|}|nt-\overline{\sum z_i}| =\frac{1}{|t|}n|t| = n $

 

Lời giải bài toán 27:

Ta chứng minh BĐT bằng quy nạp.Thật vậy,ta chỉ cần chứng minh bước quy nạp sau:

\[ \sum_{k = n^2+1}^{(n+1)^2}\left\{\sqrt{k} \right\} \le\frac{2n+1}{2} \]

 

Ta sẽ chứng minh nó bằng cách chỉ ra rằng :

\[ \left\{\sqrt{n^2+i}\right\}+\left\{\sqrt{(n+1)^2-i}\right\}\le\frac{2n+1}{2n} \]

 

Lúc này,ta có do $ \lfloor\sqrt{n^2+i}\rfloor =\lfloor\sqrt{(n+1)^2-i}\rfloor = n $ nên BĐT trên tương đương:

\[ \sqrt{n^2+i}+\sqrt{(n+1)^2-i}\le\frac{4n^2+2n+1}{2n} \]

 

Bằng tính chất hàm lõm:

\[ \sqrt{n^2+i}+\sqrt{(n+1)^2-i}\le 2\sqrt{\frac{n^2+(n+1)^2}{2}}= 2\sqrt{\frac{2n^2+2n+1}{2}} \]

 

Do đó ta chỉ cần chứng minh:

\[ 2\sqrt{\frac{2n^2+2n+1}{2}}\le\frac{4n^2+2n+1}{2n} \]

\[ 8n^2(2n^2+2n+1)\le (4n^2+2n+1)^2 \]

\[ 0\le (2n+1)^2 \]

 

Vậy ta có đpcm.

 

====================

Đề mới:

 

Bài toán 29: Trong $2013$ số thực cho sẵn bất kỳ,chứng minh rằng sẽ có ít nhất 1 cặp số $(a;b)$ thỏa mãn BĐT $\frac{|1-ab||a-b|}{(a^2+1)(b^2+1)}<\frac{1}{2012}$.

 

Bài toán 30: Cho $2n$ số thực $a_1,a_2,...,a_n,b_1,b_2,...,b_n$ thỏa mãn $1 \ge b_1 \ge b_2 \ge ...\ge b_n \ge 0$.Chứng minh rằng tồn tại 1 số tự nhiên $k \le n$ sao cho $ |a_1b_1+a_2b_2+...+a_nb_n|\le |a_1+a_2+...+a_k| $


Bài viết đã được chỉnh sửa nội dung bởi dark templar: 24-06-2013 - 13:12

"Do you still... believe in me ?" Sarah Kerrigan asked Jim Raynor - Starcraft II:Heart Of The Swarm.

#51
dark templar

dark templar

    Kael-Invoker

  • Hiệp sỹ
  • 3788 Bài viết
Bài toán 29: Trong $2013$ số thực cho sẵn bất kỳ,chứng minh rằng sẽ có ít nhất 1 cặp số $(a;b)$ thỏa mãn BĐT $\frac{|1-ab||a-b|}{(a^2+1)(b^2+1)}<\frac{1}{2012}$.

 

Bài toán 30: Cho $2n$ số thực $a_1,a_2,...,a_n,b_1,b_2,...,b_n$ thỏa mãn $1 \ge b_1 \ge b_2 \ge ...\ge b_n \ge 0$.Chứng minh rằng tồn tại 1 số tự nhiên $k \le n$ sao cho $ |a_1b_1+a_2b_2+...+a_nb_n|\le |a_1+a_2+...+a_k| $

Lời giải bài toán 29

Lời giải bài toán 30

 

Đề mới:

 

Bài toán 31: Cho hàm $f$ là 1 đơn ánh từ $\mathbb{N^*}$ đến $\mathbb{N^*}$.Chứng minh với mọi $n$ thì $ \sum_{k=1}^{n}f(k)k^{-2}\geq\sum_{k=1}^{n}k^{-1}. $

 

Bài toán 32: Tìm số nguyên $k$ nhỏ nhất thỏa $ \sqrt{1+\sqrt{2+\sqrt{3+\ldots+\sqrt{2007}}}}<k $.

 

1 BĐT mạnh hơn là $ \sqrt{k+\sqrt{k+1+\sqrt{k+2+...+\sqrt{k+n}}}}<k+1 $

-218-

 

Nhận xét

 

====================

Chú ý: Kể từ giờ,các bạn tham gia topic nếu post lời giải quá dài thì hãy đặt nó trong cặp thẻ sau:

[hide="Lời giải bài toán"]<Nội dung>[/hide] 

Kết quả :

Lời giải bài toán
để load trang topic và công thức Toán nhanh hơn và tăng thẩm mỹ cho topic.

 


Bài viết đã được chỉnh sửa nội dung bởi dark templar: 24-06-2013 - 17:19

"Do you still... believe in me ?" Sarah Kerrigan asked Jim Raynor - Starcraft II:Heart Of The Swarm.

#52
babystudymaths

babystudymaths

    Sĩ quan

  • Thành viên
  • 312 Bài viết

Bài toán 28

Bđt tương đương $\sum (\frac{a}{b}-\frac{1-a}{1-b})\geq 0\Leftrightarrow \sum \frac{a}{b(b-1)}\geq \sum \frac{b}{b(1-b)}$

Thấy nếu giả sử $a\geq b\geq c\geq d\Rightarrow a(1-a)-b(1-b)= (a-b)(1-a-b)\geq 0$ nên a,b,c,d và a(1-a),b(1-b),c(1-c),d(1-d) là các bộ đơn điệu cùng chiều,theo bđt hoán vị ta dễ dàn suy ra đ.p.c.m

Đẳng thức xảy ra khi và chỉ khi a=b=c=d.   Có thể tổng quát cho n số với cách làm tương tự như trên


TLongHV


#53
Nguyen Huy Tuyen

Nguyen Huy Tuyen

    Trung sĩ

  • Thành viên
  • 104 Bài viết

Bài 32: Ta chứng minh tổng quát $\sqrt{x+k+\sqrt{x+k+1+\sqrt{x+k+2+....+\sqrt{x+n}}}}< x+k+1$

Ta dễ dàng chứng minh được: $\sqrt{x+n}< x+n+1$

Giả sử bất đẳng đúng với $k=m+1$, bây giờ ta cần chứng minh bất đẳng thức cũng đúng với $k=m$

Ta có: $\sqrt{x+m+1+\sqrt{x+m+2+\sqrt{x+m+3+....+\sqrt{x+n}}}}< x+m+2< x^2+m^2+x+m+1$

           $\Leftrightarrow x+m+\sqrt{x+m+1+\sqrt{x+m+2+\sqrt{x+m+3+....+\sqrt{x+n}}}}< x^2+m^2+2x+2m+1=(x+m+1)^2$

           $\Leftrightarrow \sqrt{x+m+\sqrt{x+m+1+\sqrt{x+m+2+\sqrt{x+m+3+....+\sqrt{x+n}}}}}< x+m+1$

Vậy bài toán được chứng minh.


Sống đơn giản, lấy nụ cười làm căn bản !


#54
chrome98

chrome98

    Mãi Mãi Việt Nam

  • Thành viên
  • 258 Bài viết

Bài 33: Cho $xyz=1, x, y, z>0$. Chứng minh rằng:

\[ \frac{1}{x^2+x+1}+\frac{1}{y^2+y+1}+\frac{1}{z^2+z+1}\le \frac{3(x^2+y^2+z^2)}{(x+y+z)^2} \]

\[ \frac{1}{x^2+x+1}+\frac{1}{y^2+y+1}+\frac{1}{z^2+z+1}\le \frac{3(x^2y^2+y^2z^2+z^2x^2)}{(xy+yz+zx)^2} \]



#55
huyphamvan

huyphamvan

    Binh nhất

  • Thành viên
  • 25 Bài viết

Bài 33: Cho $xyz=1, x, y, z>0$. Chứng minh rằng:

$$ \frac{1}{x^2+x+1}+\frac{1}{y^2+y+1}+\frac{1}{z^2+z+1}\le \frac{3(x^2+y^2+z^2)}{(x+y+z)^2} $$

$$ \frac{1}{x^2+x+1}+\frac{1}{y^2+y+1}+\frac{1}{z^2+z+1}\le \frac{3(x^2y^2+y^2z^2+z^2x^2)}{(xy+yz+zx)^2} $$

Ta có
$$\sum {\frac{1}{{{x^2} + x + 1}}}  - 1 = \frac{{{{(x - y)}^2} + {{(y - z)}^2} + {{(z - x)}^2}}}{{2\prod {({x^2} + x + 1)} }}$$

$$\frac{{3({x^2} + {y^2} + {z^2})}}{{{{(x + y + z)}^2}}} - 1 = \frac{{{{(x - y)}^2} + {{(y - z)}^2} + {{(z - x)}^2}}}{{{{(x + y + z)}^2}}}$$
BĐT cần chứng minh tương đương với

$$\sum {\frac{1}{{{x^2} + x + 1}} \le } \frac{{3({x^2} + {y^2} + {z^2})}}{{{{(x + y + z)}^2}}}$$

$$ \Leftrightarrow \sum {\frac{1}{{{x^2} + x + 1}}}  - 1 \le \frac{{3({x^2} + {y^2} + {z^2})}}{{{{(x + y + z)}^2}}} - 1$$

$$ \Leftrightarrow \frac{{{{(x - y)}^2} + {{(y - z)}^2} + {{(z - x)}^2}}}{{2\prod {({x^2} + x + 1)} }} \le \frac{{{{(x - y)}^2} + {{(y - z)}^2} + {{(z - x)}^2}}}{{{{(x + y + z)}^2}}}$$

$$ \Leftrightarrow \left[ {\frac{1}{{{{(x + y + z)}^2}}} - \frac{1}{{2\prod {({x^2} + x + 1)} }}} \right]{(x - y)^2} + {(y - z)^2} + {(z - x)^2} \ge 0$$

BĐT trên đúng do: $$2\prod {({x^2} + x + 1)}  > {(x + y + z)^2}$$

Dấu = xảy ra $$ \Leftrightarrow x = y = z = 1$$


Bài viết đã được chỉnh sửa nội dung bởi huyphamvan: 11-04-2014 - 21:32

P.V.H
"If I feel happy, I do mathematics to become happy.
If I am happy, I do mathematics to keep happy."
(Alfred Renyi
)

"It is the peculiar beauty of this method, gentlemen, and one which endears it to the really scientific mind, that under no circumstance can it be of the smallest possible utility"
(G.-C.Rota, Indiscrete Thoughts, Birkhauser, Boston, 1977.)


#56
huyphamvan

huyphamvan

    Binh nhất

  • Thành viên
  • 25 Bài viết

Bài 33: Cho $xyz=1, x, y, z>0$. Chứng minh rằng:

$$ \frac{1}{x^2+x+1}+\frac{1}{y^2+y+1}+\frac{1}{z^2+z+1}\le \frac{3(x^2y^2+y^2z^2+z^2x^2)}{(xy+yz+zx)^2} $$

Với $a=b= \frac{1}{4}; c=16$ thì $$ \frac{1}{x^2+x+1}+\frac{1}{y^2+y+1}+\frac{1}{z^2+z+1} > \frac{3(x^2y^2+y^2z^2+z^2x^2)}{(xy+yz+zx)^2} $$


P.V.H
"If I feel happy, I do mathematics to become happy.
If I am happy, I do mathematics to keep happy."
(Alfred Renyi
)

"It is the peculiar beauty of this method, gentlemen, and one which endears it to the really scientific mind, that under no circumstance can it be of the smallest possible utility"
(G.-C.Rota, Indiscrete Thoughts, Birkhauser, Boston, 1977.)


#57
chrome98

chrome98

    Mãi Mãi Việt Nam

  • Thành viên
  • 258 Bài viết
Bài 34: Prove or disprove the following inequality: for $a,b,c>0$
\[ \sqrt[3]{3a^3+5b^3}+ \sqrt[3]{3b^3+5c^3}+  \sqrt[3]{3c^3+5a^3}\ge a+b+c+\sqrt[3]{9(a^3+b^3+c^3)} \]
 
enjoy !!!


#58
DOTOANNANG

DOTOANNANG

    Đại úy

  • ĐHV Toán Cao cấp
  • 1609 Bài viết

 



#59
DOTOANNANG

DOTOANNANG

    Đại úy

  • ĐHV Toán Cao cấp
  • 1609 Bài viết



#60
DOTOANNANG

DOTOANNANG

    Đại úy

  • ĐHV Toán Cao cấp
  • 1609 Bài viết







Được gắn nhãn với một hoặc nhiều trong số những từ khóa sau: tuyển tập, sưu tầm.

2 người đang xem chủ đề

0 thành viên, 2 khách, 0 thành viên ẩn danh